0% found this document useful (0 votes)
22 views

Sobolev

1) The document provides exercises related to Sobolev spaces. 2) The first exercise defines outward unit vectors on the boundaries of a domain and uses this to prove an integration by parts formula involving surface integrals over the boundaries. 3) The second exercise shows that the sign function sgn(x) has no weak derivative by working with distributions and showing its weak derivative is equal to 2 times the Dirac delta function, which is not in L1loc. 4) The third exercise proves that if a function u has weak derivatives v1 and v2 in overlapping open sets U1 and U2, then v1 = v2 in the overlap, and v defines the weak derivative of u over

Uploaded by

Asma Zerrouki
Copyright
© © All Rights Reserved
We take content rights seriously. If you suspect this is your content, claim it here.
Available Formats
Download as PDF, TXT or read online on Scribd
0% found this document useful (0 votes)
22 views

Sobolev

1) The document provides exercises related to Sobolev spaces. 2) The first exercise defines outward unit vectors on the boundaries of a domain and uses this to prove an integration by parts formula involving surface integrals over the boundaries. 3) The second exercise shows that the sign function sgn(x) has no weak derivative by working with distributions and showing its weak derivative is equal to 2 times the Dirac delta function, which is not in L1loc. 4) The third exercise proves that if a function u has weak derivatives v1 and v2 in overlapping open sets U1 and U2, then v1 = v2 in the overlap, and v defines the weak derivative of u over

Uploaded by

Asma Zerrouki
Copyright
© © All Rights Reserved
We take content rights seriously. If you suspect this is your content, claim it here.
Available Formats
Download as PDF, TXT or read online on Scribd
You are on page 1/ 28

2019/2020

Master 2 Recherche fondamentale en mathématiques

Sobolev spaces’s exercises

Professor : Roger Lewandowski

Written by Pierre Le Barbenchon


1 Sobolev spaces
1.1 General framework

Exercise 1. N = 2

1) Define the outward unit vector at any point x


Z
2) Define g(x)dσ(x) for g smooth
Γi

3) Prove (IP ) by using 1D integration by part and Fubini’s theorem.

y Γ = Γ1 ∪ Γ2

n
z+ Γ1
> y = f1 (x)

< y = f2 (x)

Γ2
x
a b

Solution. 1) The tangent of Γ1 on x has (1, f10 (x)) as director vector. Then, the normal vector is
(−f10 (x), 1) because the inner product has to be zero. We want to have a outward unit vector,
so we divide by the norm and find

−f 0 (x) 1
~n(1) = ( p 0 1 ,p 0 )
2
(f1 (x)) + 1 (f1 (x))2 + 1

Same method for Γ2 and we find :

−f 0 (x) 1
~n(2) = ( p 0 2 ,p 0 )
2
(f2 (x)) + 1 (f2 (x))2 + 1

1
2) We parameterize Γ1 with (x, f1 (x)) for x ∈ [a, b]. Then we use line integral.
Z Z b
g(x)dσ(x) = g(f1 (x))||f10 (x)||dx
Γ1 a
s 
dx 2 df1 (x) 2
Z b  
= g(f1 (x)) + dx
a dx dx
Z b q
= g(f1 (x)) 1 + (f10 (x))2 dx
a

Same method for Γ2 :


Z Z a q
g(x)dσ(x) = g(f2 (x)) 1 + (f20 (x))2 dx
Γ2 b

We change the bounds order because we go from b to a on the boundary Γ2 .

3) We use the Fubini theorem to split the open Ω


!
Z Z b Z f1 (x)
∂v ∂v
u(x, y) (x, y)dxdy = u(x, y) (x, y)dy dx
Ω ∂y x=a y=f2 (x) ∂y

Then we use integration by parts :


!
Z b Z f1 (x)
f (x) ∂u
= [u(x, y)v(x, y)]f12 (x) − (x, y)v(x, y)dy dx
x=a f2 (x) ∂y
!
Z b Z f1 (x)
∂u
= u(x, f1 (x))v(x, f1 (x)) − u(x, f2 (x))v(x, f2 (x)) − (x, y)v(x, y)dy dx
x=a f2 (x) ∂y

Z b
1
q
= u(x, f1 (x))v(x, f1 (x)) p 1 + (f10 (x))2 dx
x=a 1 + (f10 (x))2
Z b
1
q
− u(x, f2 (x))v(x, f2 (x)) p 1 + (f20 (x))2 dx
x=a 1 + (f20 (x))2
Z b Z f1 (x)
∂u
− (x, y)v(x, y)dydx
x=a f2 (x) ∂y
Z b q
(1)
= uvn2 1 + (f10 (x))2 dx
x=a
Z a q
(2)
+ uvn2 1 + (f20 (x))2 dx
x=b
Z b Z f1 (x)
∂u
− (x, y)v(x, y)dydx
x=a f2 (x) ∂y
Z Z Z
(1) (2) ∂u
= uvn2 dσ + uvn2 dσ − (x, y)v(x, y)dydx
Γ1 Γ2 Ω ∂y
Z Z
∂u
= uvn2 dσ − (x, y)v(x, y)dydx
Γ Ω ∂y

2
We finally find the (IP ) formula. (We justify the Fubini Theorem because u, v ∈ C 1 , then
∂v
they are continuous on Ω compact, then u ∈ L1 (Ω))
∂y

Exercise 2. Let Ω =] − 1; 1[⊂ R.


x
 if x 6= 0
sgn(x) = |x| ∈ L∞ (Ω)
0 if x = 0

Show that x 7→ sgn(x) has no weak derivative.

Solution. We work in the distributions. For all ϕ ∈ D(] − 1, 1[),

hsgn0 , ϕi = −hsgn, ϕ0 i
Z 0 Z 1
0
=+ ϕ − ϕ0
−1 0
= [ϕ]0−1 − [ϕ]10
= 2ϕ(0)
= h2δ0 , ϕi

Then sgn0 = 2δ0 . But this distribution isn’t in L1loc , so x 7→ sgn(x) has no weak derivative.

To prove that the dirac δ0 is not in L1loc , we work for a contradiction. Let f ∈ L1loc such that
Z 1
∀ϕ ∈ D(] − 1, 1[), ϕ(0) = fϕ
−1

We use a sequence ϕn ∈ D(] − 1, 1[) such that supp(ϕn ) ⊂ [− n1 , n1 ], ϕn (0) = 1 and the supremum
of each ϕn equal to 1. R1
Then we have by the dominated convergence theorem that −1 f ϕn goes to 0 because we can
dominate |1[− 1 , 1 ] f ϕn | by |f |1[−1,1] which is in L1 .
n n
We have a contradiction because ϕn (0) = 1 for all n ∈ N.

Exercise 3. Let U1 and U2 be two opensets in Rn such that U1 ∩ U2 6= ∅. Let Ω = U1 ∪ U2 ,


α = (α1 , . . . , αn ) a multiindex, and u ∈ L1loc (Ω)
Assume that u has a weak derivative v1 = Dα u in U1 and v2 = Dα u in U2 .

1) Prove that v1 = v2 in U1 ∩ U2
(
v1 in U1
2) Let v = .
v2 in U2
Prove that Dα u exists in Ω and Dα u = v.

3
Solution. 1) Soit ϕ ∈ D(U1 ∩ U2 ),
Z Z
hu, Dα ϕi = uDα ϕ = − v1 ϕ
U1 ∩U2 U1 ∩U2

and Z Z
α α
hu, D ϕi = uD ϕ = − v2 ϕ
U1 ∩U2 U1 ∩U2
R
Then U1 ∩U2 (v2 − v1 )ϕ = 0 for all ϕ ∈ D(U1 ∩ U2 ). Hence, we have v1 = v2 in U1 ∩ U2 . We
just use the injectivity of (
L1loc (Ω) → D0 (Ω)
f 7→ Tf

2) Let ϕ ∈ D(Ω). We take a unit partition of Ω = U1 ∪ U2 relatively to supp(ϕ). Then


we have θ1 , θ2 ∈ D(Ω) with θ1 + θ2 = 1, 0 6 θ1 , θ2 6 1, supp(θ1 ) ⊂ U1 ∩ supp(ϕ) and
supp(θ2 ) ⊂ U2 ∩ supp(ϕ)
We have ϕ = θ1 ϕ + θ2 ϕ

hDα u, ϕi = hDα u, θ1 ϕi + hDα u, θ2 ϕi


Z Z
= v 1 θ1 ϕ + v2 θ2 ϕ because suppθ1,2 ⊂ U1,2
U1 U2
Z Z
= vθ1 ϕ + vθ2 ϕ
ZU1 U2
Z
= vθ1 ϕ + vθ2 ϕ by definition of θ1 and θ2 supports
U1 ∪U2 U1 ∪U2
Z
= v(θ1 + θ2 )ϕ
U1 ∪U2
= hv, ϕi

Hence Dα u exists in Ω and we have Dα u = v in Ω.

1.2 Definition and basic properties of W m,p (Ω)

Exercise 4. Let Ω = B(0, 1) ⊂ R2 .



Ω →R
u:  1 
x 7→ ln ln
|x|

Prove that u ∈ H 1 (Ω) (but u ∈


/ C 0 (Ω))

4
Remark :
I just show the result for H 1 (B(0, 21 )) because I didn’t succeed for u0 ∈ L2 (B(0, 1)) (since it’s
not true), but it keeps the spirit of the exercise because we have a function in H 1 (B(0, 21 )) which
is not in C 0 (B(0, 12 )).

u(x, y)

−1
x
0
1
−1

/ C 0 (Ω). And we can’t find a continuous


Solution. The function u is not continuous in (0, 0), so u ∈
representative of u (somebody continuous equal to u almost everywhere).

∂ ∂
We want to prove that u ∈ L2 (Ω) and u, u ∈ L2 (Ω).
∂x ∂y

Step 1 : Let start with u ∈ L2 (Ω).


Z Z 1 Z 2π
2 1 2 
|u(x)| dx = ln
ρdθdρ ln
Ω ρ=0 θ=0 ρ
Z 1 
1 2
= 2π ln ln ρdρ
0 ρ

while doing a change of coordinates and seeing that ln ρ1 is non negative for ρ ∈]0, 1[.
We use the inequality ln(x) 6 x − 1 for x > 0.
Z Z 1 1 2
2
|u(x)| dx = 2π ln − 1 ρdρ
Ω 0 ρ
Z 1
= 2π (− ln(ρ) − 1)2 ρdρ
0
Z 1
= 2π (ln(ρ) + 1)2 ρdρ
0

5
We use an other change of coordinates : ρ = exp(−x)
Z Z +∞
2
|u(x)| dx = 2π (1 − x)2 e−2x dx < +∞
Ω 0
We know that x 7→ (1 − x)2 e−2x is continuous on R+ and summable in +∞ and 0. Hence,
u ∈ L2 (Ω)
Step 2 : We want to show that ∇u !! ∈ L2 (B(0, 21 )) (B(0, 21 ) as I said in the previous remark).
1
Let u(x, y) = ln ln p for all (x, y) ∈ B(0, 12 ).
x + y2
2

Assume for the moment that the weak derivative is equal to the classical derivative.
We have
∂ −2x 1 x
u(x, y) = 1
p .  = p
∂x √ 2( x2 + y 2 )3/2 ln √ 1 (x2 + y 2 ) ln( x2 + y 2 )
2
x +y 2
2
x +y 2

and
∂ y
u(x, y) = p
∂y (x + y ) ln( x2 + y 2 )
2 2

Let compute the L2 norm.


Z Z
∂ 2 ∂
u(x, y) dxdy + u(x, y)2 dxdy
B(0,1/2) ∂x B(0,1/2) ∂y
x2 y2
Z
= 2
p + p dxdy
B(0,1/2) (x2 + y 2 )2 ln ( x2 + y 2 ) (x2 + y 2 )2 ln2 ( x2 + y 2 )
Z
1
= 2
p dxdy
B(0,1/2) (x + y ) ln ( x2 + y 2 )
2 2
Z 2π Z 1/2
1
= 2 2
ρdρdθ
θ=0 ρ=0 ρ ln ρ
Z 1/2
1
=2π 2 dρ we use the change of variables ρ = ex
ρ=0 ρ ln ρ
Z − ln(2) x
e
=2π x 2
dx because dρ = ex dx
x=−∞ e x
Z − ln(2)
1
=2π 2
dx < +∞
x=−∞ x

We see in the last line why I work in B(0, 12 ).


It proves that ∇u ∈ L2 (B(0, 21 )) and then u ∈ H 1 (B(0, 12 )).
Step 3 : We have to justify that the weak derivative is equal to the classical derivative.
Let take ϕ ∈ D(B(0, 1)),
∂ ∂
h u, ϕi = −hu, ϕi
∂x Z ∂x

=− u ϕ
B(0,1) ∂x
Z

= −lim u ϕ
→0 B(0,1)\B(0,) ∂x

6
We can apply the Green Formula
Z Z Z Z
∂ ∂
u ϕ=− uϕ + uϕnx dσ + uϕnx dσ
B(0,1)\B(0,) ∂x B(0,1)\B(0,) ∂x S(0,) S(0,1)

But ϕ ∈ D(B(0, 1)), so ϕ vanishes on S(0, 1)1 .


R
And S(0,) uϕnx dσ 6 kuk∞,S(0,) kϕk∞,S(0,) 2π because 2π is the perimeter of a cercle with
radius 1. This quantity goes to zero when  goes to zero.
It follows that
Z
∂ ∂
h u, ϕi = −lim − uϕ
∂x →0 B(0,1)\B(0,) ∂x
Z

= lim uϕ
→0 B(0,1)\B(0,) ∂x
Z
x
= lim p ϕ(x, y)dxdy
→0 B(0,1)\B(0,) (x + y ) ln( x2 + y 2 )
2 2
Z
x
= p ϕ(x, y)dxdy
B(0,1) (x + y ) ln( x2 + y 2 )
2 2

while using the dominated convergence theorem.



We can do the same method for u, then the weak derivative is equal to the classical one.
∂y

Exercise 5. Let (ri )i be a countable and dense set in B(0, 1) ⊂ RN , α > 0.


+∞
X 1
u(x) = |x − ri |−α
2i
i=0

For p > 1, prove that u ∈ W 1,p (B(0, 1)) for all α < α0 with α0 = α0 (N, p) to be calculated.
1
(Hint : start by studying x 7→ )
|x|α

Remark :
I will prove the result for α0 = Np − 1 and that is a sufficient condition as the exercise asks.
But it’s a necessary and sufficient condition.

Solution. Z
1
Step 1 : We want to show that α
dx < ∞ if and only if α < N . To compute
Z B(0,1) |x|
1
α
dx, we use a spherical coordinates change in dimension N .
B(0,1) |x|
1
S(0, 1) represents the sphere of radius 1

7
x1 = ρ cos(θ1 )
x2 = ρ sin(θ1 ) cos(θ2 )
x3 = ρ sin(θ1 ) sin(θ2 ) cos(θ3 )
.. ..
. .
xN −1 = ρ sin(θ1 ) . . . sin(θN −2 ) cos(θN −1 )
xN = ρ sin(θ1 ) . . . sin(θN −2 ) sin(θN −1 )

The spherical volume element is

dV = ρN −1 sin(θ1 )N −2 sin(θ2 )N −3 . . . sin(θN −3 )2 sin(θN −2 )dρdθ1 . . . dθN −1

Z
1
dx
B(0,1) |x|α
Z1 Z π Z 2π
1
= dV
ρ=0 θ1,...,N −2 =0 θN −1 =0 ρα

But the following quantity is finite, let call it K.


Z π Z 2π
K= sin(θ1 )N −2 sin(θ2 )N −3 . . . sin(θN −3 )2 sin(θN −2 )dθ1 . . . dθN −1
θ1,...,N −2 =0 θN −1 =0

Then the computation gives

Z
1
dx
B(0,1) |x|α
Z 1
1 N −1
=K ρ dρ
ρ=0 ρα
Z 1
1
=K dρ
ρ=0 ρα−N +1

The integral converges if and only if α − N + 1 < 1. Then


Z
1
α
dx < ∞ if and only if α < N
B(0,1) |x|

1
Step 2 : Let introduce fα : RN → R such that f (x) = . Thank to the step 1, we have
|x|α
fα ∈ Lp (B(0, 1)) if and only if pα < N .
N
So we have fα ∈ Lp (B(0, 1)) if and only if α < .
p

8
Step 3 : We want to show that u ∈ Lp (B(0, 1)) if α < Np .
We start by using the Fatou’s Lemma with the functions
n
!p
X 1
hn (x) = |x − ri |−α
2i
i=0

We have Z Z
lim inf hn 6 lim inf hn

While passing to the power p1 , we have


Z 1 Z 1
p p
lim inf hn 6 lim inf hn

n
!p !1 n
!p !1
Z p Z p
X 1 X 1
lim inf |x − ri |−α dx 6 lim inf |x − ri |−α dx
2i 2i
i=0 i=0
Z +∞
!p ! p1 n
X 1 X 1
|x − ri |−α dx 6 lim inf fα (. − ri )
2i 2i
i=0 i=0 p
n
X 1
kukp 6 lim inf kfα (. − ri ) kp
2i
i=0
+∞
X 1
kukp 6 kfα (. − ri ) kp
2i
i=0

We used the Minkowski inequality to get out the sum of the norm Lp .
But thanks to step 2, while using a translation of ri , we have
N
kfα (. − ri )kp < +∞ if and only if α <
p
And all those quantities are dominated by the same function g = fα on B(0, 2), so
+∞
X 1
kfα (. − ri ) kp < +∞
2i
i=0

N
Then u ∈ Lp if α < .
p
Step 4 : We want to show that u0 ∈ Lp (B(0, 1)).
We start to find the derivative of fα (. − r) for r = (r1 , . . . , rN ) ∈ B(0, 1). For the same reasons
of the Exercise 4 Step 3, the weak derivative is equal to the classical derivative.
∂ ∂ 1
fα (. − r) =
∂xk ∂xk ( (xj − rj )2 )α/2
P
 α −1
− α2 2(xk − rk ) (xj − rj )2 2
P
=
|x − r|2α
−α(xk − rk )
=
|x − r|α+2

9
∂ α N
Then we have fα (. − r) < . And it’s in Lp if α + 1 < p. (because of the Step 2)
∂xk |x − r|α+1

We want to show that


+∞
! +∞
∂ X 1 X 1 ∂
i
fα (. − ri ) = fα (. − ri ) (1)
∂xk 2 2i ∂xk
i=0 i=0

For all ϕ ∈ D(B(0, 1)),


+∞
* ! + * +∞ +
∂ X 1 X 1 ∂
fα (. − ri ) , ϕ = fα (. − ri ), ϕ
∂xk 2i 2i ∂xk
i=0 i=0
Z X+∞
1 ∂
= fα (. − ri ) ϕ
2i ∂xk
i=0
+∞ Z
X 1 ∂
= i
fα (. − ri ) ϕ
2 ∂xk
i=0
+∞  
X 1 ∂
= fα (. − ri ), ϕ
2i ∂xk
i=0
+∞  
X 1 ∂
= fα (. − ri ), ϕ
2i ∂xk
i=0

Z X
The justification of the change of and is because of the dominated convergence theorem.

n +∞
X 1 ∂ X 1 ∂
i
fα (. − ri ) ϕ −−−→ i
fα (. − ri ) ϕ a.e.
2 ∂xk n→∞ 2 ∂xk
i=0 i=0

and
n ∞
X 1 ∂ ∂ X 1
fα (. − ri ) ϕ 6 ϕ 1Supp ϕ fα (. − ri )
2i ∂xk ∂xk ∞ 2i
i=0
|i=0 {z }
∈Lp (Step 3)
| {z }
∈L1 (Supp ϕ)

Then it justifies the egality (1) and so, thanks the same methods that Step 3 and the beginning
of Step 4, we have
+∞
!
∂ X 1 N
fα (. − ri ) < +∞ if and only if α < −1
∂xk 2i p
i=0 p

Step 5 : Conclusion
N
Thanks to Step 3 and 4, we have u ∈ W 1,p (B(0, 1)) for all α < α0 with α0 = − 1.
p

10
1.3 Duality spaces W −m,p

Exercise 6. Let α = (α1 , . . . , αN ), x = (x1 , . . . , xN ). Case of Ω = RN .

S := {u ∈ L2 (RN ) such that xα Dβ u ∈ L2 (RN )∀α, β multiindex}

Show that
H m (RN ) = {u ∈ S 0 , such that (1 + |ξ|2 )m/2 û ∈ L2 (RN )}
where û is the Fourier transform of u.

Solution. We recall that

H m (RN ) := {u ∈ L2 (RN ), Dα u ∈ L2 (RN ), ∀|α| 6 m}

Step 1 : We wantPto remark that F(Dα u)(ξ) = i|α| ξ α F(u)(ξ) for u ∈ S, α = (α1 , . . . , αN ) ∈ NN ,
αN
ξ ∈ RN with |α| = i αi and ξ α = ξ1α1 . . . ξN .
For ϕ ∈ S,

hF(Dα u), ϕi = hDα u, ϕi


= (−1)|α| hu, Dα (Fϕ)i
= (−1)|α| hu, (−i)|α| ξ α Fϕi
= i|α| ξ α hu, Fϕi
= i|α| ξ α hFu, ϕi

Then we have F(Dα u)(ξ) = i|α| ξ α F(u)(ξ).

Step 2 : We want to prove the direct inclusion.


Let u ∈ H m (RN ), then we have by Fourier–Plancherel, F(Dα u) ∈ L2 for all α such that |α| 6 m.
But Dα u ∈ S 0 , so i|α| ξ α û ∈ L2 (by Step 1).

Z  2 Z
m
2
(1 + |ξ| ) û(ξ)
2 dξ = (1 + |ξ|2 )m û2 (ξ)dξ
RN RN
Z m  
X m
= |ξ|2k û2 (ξ)dξ
RN k=0 k
m  Z
X m
= |ξ|2k û2 (ξ)dξ because k 6 m
k N
k=0 |R {z }
<∞

Since u is in S 0 (because L2 ⊂ S 0 ), then

u ∈ {u ∈ S 0 , such that (1 + |ξ|2 )m/2 û ∈ L2 (RN )}

11
Step 3 : We want to prove the other inclusion.
Let u ∈ S 0 such that (1 + |ξ|2 )m/2 û ∈ L2 (RN ).
Let α ∈ NN such that |α| 6 m.
We know that
(1 + |ξ|2 )m û2 (ξ) ∈ L1 (RN ),
then
(1 + |ξ|2 )α û2 (ξ) ∈ L1 (RN ).
It follows that
|ξ|2α û2 (ξ) ∈ L1 (RN ),
then
i|α| ξ α û(ξ) ∈ L2 (RN ).
We recognize the Fourier Transform of Dα u, and, by Fourier–Plancherel, we conclude that
Dα u ∈ L2 (RN ).
Therefore u ∈ H m (RN ).

1.4 Study of W 1,p (Ω)

2
( Ω = {(x, y), 0 < |x| < 1, 0 < y < 1} ⊂ R .
Exercise 7. Let
1 if x > 0
Let u(x, y) =
0 if x < 0

1) Show that u ∈ W 1,p (Ω), ∀p > 1

2) Show there is  > 0, such that there is no function φ ∈ C 1 (Ω) such that ku − φk1,p < .

3) What’s up ?

u(x, y)

x
−1 0 1

12
Solution. 1) Step 1 : We want to show that u ∈ Lp (Ω).
Z 0 Z 1 Z 1 Z 1 Z 1 Z 1
p p
|u(x, y)| dydx + |u(x, y)| dydx = 1dydx = 1
x=−1 y=0 | {z } x=0 y=0 | {z } x=0 y=0
=0 =1

Then u ∈ Lp (Ω).

Step 2 : We want to show that Du ∈ Lp (Ω).


Let ϕ ∈ D(Ω), then since the support of ϕ is compact in Ω, we have ϕ(0, y) = ϕ(1, y) = 0 for
all y ∈ [0, 1] and ϕ(x, 1) = ϕ(x, 0) = 0 for all x ∈ [0, 1].

h∂y u, ϕi = −hu, ∂y ϕi
Z
= − u∂y ϕ

Z 1 Z 1
=− ∂y ϕ(x, y)dydx
x=0 y=0
Z1
=− (ϕ(x, 1) − ϕ(x, 0))dx
x=0
=0

h∂x u, ϕi = −hu, ∂x ϕi
Z
= − u∂x ϕ

Z 1 Z 1
=− ∂x ϕ(x, y)dxdy
y=0 x=0
Z 1
=− (ϕ(1, y) − ϕ(0, y))dy
y=0
=0

Then ∂x u(x, y) = ∂y u(x, y) = 0 for all (x, y) ∈ Ω.


So ∂x u, ∂y u ∈ Lp (Ω) and therefore u ∈ W 1,p (Ω).

2) We want to show that there exists ε such that for all φ ∈ C 1 (Ω),

ku − φk1,p,Ω > ε.

We work for a contradiction. We suppose that


1
∀n > 0, ∃φn ∈ C 1 (Ω) s.t. ku − φn k1,p,Ω 6
n

13
So we have a sequence (φn )n such that

W 1,p (Ω)
φn −−−−−→ u
n→∞

It means that
Lp (Ω) Lp (Ω)
φn −−−−→ u and ∇φn −−−−→ 0 (2)
n→∞ n→∞

Because ∂x u(x, y) = ∂y u(x, y) = 0 for all (x, y) ∈ Ω.


By the Lebesgue Inverse Theorem, we have a subsequence, still called φn , that converges to u
almost everywhere in Ω.
We have for almost everywhere y ∈]0, 1[, x1 < 0, x2 > 0

φn (x1 , y) −−−→ u(x1 , y) = 0
n→∞
φn (x2 , y) −−−→ u(x2 , y) = 1
n→∞

Then

|φn (x1 , y) − φn (x2 , y)| −−−→ |u(x1 , y) − u(x2 , y)| = 1 (3)


n→∞

On the other hand,


Z 1
|φn (x1 , y) − φn (x2 , y)| 6 |Dφn (tx1 + (1 − t)x2 , y).(x1 − x2 , 0)|dt
0
Z 1
6 kDφn (tx1 + (1 − t)x2 , y)kk(x1 − x2 , 0)kdt by Cauchy–Schwarz
0
Z 1 1/p
6 k(x1 − x2 , 0)k kDφn (tx1 + (1 − t)x2 , y)kp dt by Hölder
0

We use a convexity inequality


p/2
a2 + b2 xp + y p

6
2 2

In other words, p
(a2 + b2 )p/2 6 2 2 −1 (ap + bp )

Z 1
kDφn (tx2 + (1 − t)x2 , y)kp dt (4)
0
Z 1
= (∂x φ2n (tx1 + (1 − t)x2 , y) + ∂y φ2n (tx1 + (1 − t)x2 , y))p/2 dt (5)
0
Z 1
6C ∂x φpn (tx1 + (1 − t)x2 , y) + ∂y φpn (tx1 + (1 − t)x2 , y)dt (6)
0

14
We use Fatou to have
Z 1 Z 1
lim inf |φn (x1 , y) − φn (x2 , y)|dy 6 lim inf |φn (x1 , y) − φn (x2 , y)|dy (7)
0 0
| {z }
=1 by (3)

By (6),
Z 1 Z 1Z 1
|φn (x1 , y) − φn (x2 , y)|dy 6 C ∂x φpn (tx1 + (1 − t)x2 , y) + ∂y φpn (tx1 + (1 − t)x2 , y)dtdy
0 Z0 0
6C k∇φn kp −−−−−→ 0 because of (2)
A n→+∞

where A = {(x, y), x1 < x < x2 , 0 < y < 1}


Therefore, by (7), we have a contradiction.
To conclude, we have that there exists ε such that for all φ ∈ C 1 (Ω),
ku − φk1,p,Ω > ε.

3) We have a theorem that says : If Ω is of class C 1 and u ∈ W 1,p (Ω), then there exists a sequence
(un )n of functions in D(RN ) such that
W 1,p (Ω)
un |Ω −−−−−→ u
n→∞

The problem is that, in our exercise, Ω is not of class C 1 and then we can’t use that theorem.

1.4.1 Density results

Exercise 8. 1) Show that ∀v ∈ H m (RN ), ∀m > 1,

kv ? ρ − vkm−1,2 6 Ckvkm,2

2) Show that ∀v ∈ H m (RN ), ∀k > 0,

Cm,k
kv ? ρ km+k,2 6 kvkm,2
k

3) Show that ∀v ∈ H m (RN ), ∀k > 0, ∀|α| 6 k

Ck
kρ ? Dα vk0,∞ 6 N/2+k
kvk0,2


Remark :
In this exercise, for the purposes of notation, I will write sometimes kf (x)kL2 instead of kf kL2
or kf (·)kL2 . It is just because I want to mention the variables sometimes.
Moreover, there is a constant while using Fourier Plancherel due to my definition of the Fourier
transform, but I will never put it (as it was 1). It doesn’t matter because it will be in the constant
C of the exercise.

15
Solution. 1) ∀|α| 6 m − 1, ∀v ∈ H m (RN )

kDα (v ? ρ − v)kL2 = kF(Dα (v ? ρ − v))kL2 with Fourier Plancherel


= kξ α F(v ? ρ − v)kL2 by properties of F
= kξ α F(v)(F(ρ ) − 1)kL2 because of the convolution product

Z
F(ρ )(ξ) = ρ (x)e−iξx dx
RNZ
1 x
= ρ e−iξx dx
N RN 
Z
1
= N ρ(u)e−iξu N du change of variable x = u
 R N

= F(ρ)(ξ)

So we have

F(ρ )(ξ) = F(ρ)(ξ) (8)


R
We remark that 1 = F(ρ)(0) because RN ρ(x)dx = 1.
Now we use the mean value theorem in RN , we have

∃θ ∈]0, 1[, s.t. F(ρ ) − 1 = F(ρ)(ξ) − F(ρ)(0) = DF(ρ)(ξθ).(ξ)

But DF(ρ)(ξθ).(ξ) is bounded by C||ξ where C is a constant.2


To conclude we have

kDα (v ? ρ − v)kL2 6 Ckξ α ξF(v)kL2


6 Ckξ α+1 F(v)kL2
6 CkF(Dα+1 v)kL2
6 CkDα+1 vkL2
6 Ckvkm,2 because |α| + 1 6 m

So we have the result. We just used the notation α + 1, we have to understand that for
instance we write the mean value theorem for the first derivative, then the “+1” is on the first
component of α.
2
Indeed, DF(ρ) = F(xρ) is bounded, because it’s the Fourier transform of a C0∞ function, so we can bound it by
the L1 −norm.

16
2) ∀|α| 6 m, ∀v ∈ H m (RN ), ∀|β| 6 k

kDα+β (v ? ρ )kL2 = kF(Dα+β (v ? ρ ))kL2 with Fourier Plancherel


= kξ α+β F(v ? ρ )kL2 by properties of F
= kξ α+β F(v)F(ρ )kL2 by properties of F and convolution product
(ξ)β
= kξ α F(v) F(ρ)(ξ)kL2 because (8)
|β|
1
= kF(Dα v)F(Dβ ρ)(ξ)kL2 because |β| 6 k
k
C
6 k kF(Dα v)kL2

C
6 k kDα vkL2

C
6 k kvkm,2 because |α| 6 m

So we have the result. We just used C the bound of F(Dβ ρ) that exists since ρ ∈ C ∞ .

3) First we will use the following inequality

∀f, g ∈ L2 , |f ? g(x)| 6 kf kL2 kgkL2

To prove that inequality, we use the definition of the convolution product and the Cauchy–
Schwarz inequality.
So we have, for |α| 6 k,

|ρ ? Dα v(x)| = |Dα (ρ ) ? v(x)| (9)


α
6 kD ρ kL2 kvk0,2 (10)

Ck
We want to prove that kDα ρ kL2 6 N/2+k
.

Z
kDα (ρ )k2L2 = Dα (ρ )(x)2 dx
ZRN

= α (ρ )(ξ)2 dξ by Fourier Plancherel


D\ 
N
ZR
= ξ 2α ρb (ξ)2 dξ
N
ZR
= ξ 2α ρb(ξ)2 dξ because (8)
RN
u2α
Z
1
= 2|α|
ρb(u)2 N du because of the change of variable ξ = u
RN  
1
= kuα F(ρ)(u)k2L2
2k N

17
So we have
1
kDα (ρ )kL2 = kF(Dα ρ)kL2
k N/2
1
6 N/2+k kDα ρkL2

1
6 N/2+k sup kDα ρkL2
 |α|6k
| {z }
Ck

Then, since (10) and while passing to the supremum, we have the result
Ck
kρ ? Dα vkL∞ 6 N/2+k
kvkL2 .


1.4.2 About traces

Exercise
P 9. Let ~u = (u1 , . . . , uN ) ∈ D(Ω)N with Ω bounded of class C 1 . We recall that div ~u =
i Di ui . We define
p
Hdiv (Ω) = {~u ∈ Lp (Ω)N , div ~u ∈ Lp (Ω)}
p
1) Show that Hdiv (Ω) is a Banach space.

2) Prove that ~u.~n|Γ can be defined in an appropriate space to be determined. (~u.~n|Γ = “normal
trace of u”)

p
Solution. 1) We put on Hdiv (Ω) the norm
N
X
k~ukdiv := kui kLp (Ω) + kdiv ~ukLp (Ω)
i=1

p
We can easily check that (Hdiv (Ω), k · kdiv ) is a normed vector space.
We want to prove now that it is complete.
p
Let (~un ) be a Cauchy sequence in Hdiv (Ω).

∀ > 0, ∃n0 > 0, ∀n, p > n0 , k~un − ~up kdiv 6 

By definition of k · kdiv and because (Lp , k · kLp ) is complete, we have


Lp
un,i −−−→ ui ∀i ∈ {1, · · · , N }
n→∞

and
Lp
div ~un −−−→ v
n→∞

18
Then we have by weak convergence,
D0
un,i * ui weakly ∀i ∈ {1, · · · , N }
n→∞

and so
D0
Di un,i * Di ui weakly ∀i ∈ {1, · · · , N }
n→∞

It follows that
D0
div ~un * div ~u weakly
n→∞

Lp
By limit uniqueness and div ~un −−−→ v, we have v = div ~u.
n→∞
Hence, we have
Lp
un,i −−−→ ui ∀i ∈ {1, · · · , N }
n→∞

and
Lp
div ~un −−−→ div ~u
n→∞
p p
which prove that ~un goes to ~u in Hdiv (Ω) and that ~u ∈ Hdiv (Ω).

2) I am sorry but I didn’t find/take the time to thing about that question...

1.5 Sobolev compact embeddings

Exercise 10. Let Ω bounded of class C 1 , 1 6 p < +∞.


Show that Z Z 1/p
p p
N : u 7→ |∇u| + |tr(u)|
Ω Γ

is a norm over W 1,p (Ω), equivalent to k.k1,p,Ω .

19
Solution. Step 1 : We want to show that N is a norm over W 1,p (Ω).

• absolutely homogeneous: For all u ∈ W 1,p (Ω), for all λ ∈ R,


Z Z 1/p
p p
N (λu) = |∇(λu)| + |tr(λu)|
Ω Γ
Z Z 1/p
p p p p
= |λ| |∇(u)| + |λ| |tr(u)|
Ω Γ
 Z Z 1/p
= |λ|p |∇(u)|p + |tr(u)|p
Ω Γ
Z Z 1/p
p p
= |λ| |∇(u)| + |tr(u)|
Ω Γ
= |λ|N (u)

• point-separating: If u = 0 then N (u) = 0. Conversely, for all u ∈ W 1,p (Ω) such that N (u) = 0.
Then, we have Z Z
p p
k∇ukLp = |∇u| = 0 and |tr(u)|p = 0
Ω Γ

Therefore, we have tr(u(x)) = 0 for all x ∈ Γ, so u ∈ W01,p (Ω).


Then by Poincaré inequality, we have

kukLp 6 Ck∇ukLp

So we have kukLp = 0, and u = 0 because k.kLp is a norm.

• triangle inequality: For all u, v ∈ W 1,p (Ω),


 1
N (u + v) = k∇(u + v)kpLp (Ω) + ktr(u + v)kpLp (Γ)
p

p p  1
6 k∇ukLp (Ω) + k∇vkLp (Ω) + ktr(u)kLp (Γ) + ktr(v)kLp (Γ) p

We have just used the triangle inequality for k.kLp .


 1  1
N (u + v) 6 k∇ukpLp (Ω) + kvkpLp (Ω) + ktr(u)kpLp (Γ) + ktr(v)kpLp (Γ)
p p

6 N (u) + N (v)

We used the Minkowski discrete inequality.


!1 !1 !1
X p X p X p
p p p
|xi + yi | 6 |xi | + |yi |
i i i

For x1 = k∇ukLp , x2 = ktr(u)kLp and y1 = k∇vkLp , y2 = ktr(v)kLp

Therefore, N is well a norm over W 1,p (Ω).

20
Step 2 : We want to show that there exists C1 such that N (·) 6 C1 k · k1,p,Ω .
We will use the continuity of the trace,

i.e. ∀u ∈ W 1,p (Ω), ktr(u)kLp (Γ) 6 Ckuk1,p,Ω

Then we can prove the wanted inequality. For all u ∈ W 1,p (Ω),

N (u)p = k∇ukpLp (Ω) + ktr(u)kpLp (Γ)


6 k∇ukpLp (Ω) + C p kukp1,p,Ω)
6 kukp1,p,Ω + C p kukp1,p,Ω)
6 C 0 kukp1,p,Ω

Then, by passing to the power p1 , we have the result.

Step 3 : We want to show that there exists C2 such that k · k1,p,Ω 6 C2 N (·).
We work for a contradiction. Suppose that we have for all n ∈ N, un ∈ W 1,p (Ω) such that

kun kLp (Ω) + k∇un kLp (Ω) > n k∇un kLp (Ω) + ktr(un )kLp (Γ) (11)
un
Let consider vn = . So we have
kun k1,p,Ω

kvn k1,p,Ω = 1 (12)

The inequality (11) becomes


1
k∇vn kLp (Ω) + ktr(vn )kLp (Γ) 6
n
Then we have
Lp (Ω) Lp (Γ)
∇vn −−−−→ 0 and tr(vn ) −−−→ 0 (13)
n→∞ n→∞

We use the Poincaré Wirtinger inequality to have

kvn − vn kLp (Ω) 6 Ck∇vn kLp (Ω) −−−→ 0


n→∞
Z
1
where vn = vn . We can remark that
|Ω| Ω

1 C C
|vn | 6 kvn kL1 (Ω) 6 kvn kLp (Ω) 6 because (12).
|Ω| |Ω| |Ω|

Hence, (vn ) is bounded and we can extract a subsequence, still called (vn ), that goes to a constant c.
Lp (Ω) C
By the dominated convergence theorem, we have vn −−−−→ c because Ω is bounded then |vn | 6
n→∞ |Ω|
that is L1 (Ω).
Lp (Ω)
But vn − vn −−−−→ 0, hence
n→∞
Lp (Ω)
vn −−−−→ c
n→∞

21
By the continuity of the trace, we have
Lp (Γ)
tr(vn ) −−−→ tr(c) = cσ(Γ) = 0 because of (13)
n→∞

where σ(Γ) est the measure of Γ. It follows that c = 0.


We conclude because
Lp (Ω) Lp (Ω)
vn −−−−→ 0 and ∇vn −−−−→ 0
n→∞ n→∞
So
W 1,p (Ω)
vn −−−−−→ 0
n→∞

It is a contradiction with (12).

/ L∞ (RN ).
Exercise 11. Find an example of u ∈ W 1,N (RN ) such that u ∈

Solution. Let take  α


1
u(x) = ln
|x|
/ L∞ . We want to find a condition on
. We can notice that u is not bounded around zero, then u ∈
α such as u(x) ∈ W 1,N (BRN (0, 12 )).
Step 1 : We want to show that u ∈ LN (BRN (0, 21 )).

1 αN
Z Z  
N
|u| = ln dx
B(0,1/2) B(0,1/2) |x|
Z 1/2  αN
1
=K ln ρN −1 dρ as the change of variable in Exercise 5 Step 1
0 ρ
Z +∞
=K xαN e−x(N −1) e−x dx by the change of variable ρ = e−x
ln(2)
Z +∞
αN −xN
=K x
| {ze } dx < +∞
ln(2)
1
!
=O
x2

Then we have the result that u ∈ LN (BRN (0, 21 )).


Step 2 : We want to show that u ∈ LN (BRN (0, 12 )) for all i ∈ J1, N K.
∂xi
Let take i ∈ J1, N K.

We start to find u. For the same reasons of the Exercise 4 Step 3, the weak derivative is
∂xi
equal to the classical derivative.
 α  α−1
∂ 1 1 −1 2xi
ln = α ln |x|
∂xi |x| |x| 2 |x|3
 α−1
1 xi
= −α ln
|x| |x|2

22
N N
1 α−1 xi
Z Z  

u = α ln dx
B(0,1/2) ∂xi B(0,1/2) |x| |x|2
Z  N (α−1)
N 1 1
6 α ln N
dx
B(0,1/2) |x| |x|
Z 1/2  N (α−1)
1 1 N −1
6K ln ρ dρ
0 ρ ρN
Z +∞
1
6K xN (α−1) −x e−x dx
ln(2) e
Z +∞
6K xN (α−1) dx
ln(2)

And x 7→ xN (α−1) is integrable on [ln(2), +∞[ if and only if N (α − 1) < −1. In other words, we
1
take α < 1 − .
N
1
It follows that, for α < 1 − , we have u ∈ W 1,N (B(0, 1/2)).
N

Step 3 : We want to extend u to be in W 1,N (RN ).


Let take a function ϕ ∈ C0∞ (RN ) such that the support of ϕ is in B(0, 1/2) and ϕ = 1 on a
neighboorhood of zero.

Then uϕ is in W 1,N (RN ) because

u ∈ W 1,N (B(0, 1/2)) and supp(ϕ) ⊂ B(0, 1/2).

And uϕ is not in L∞ (RN ) since the problem of u in zero.

2 Elliptic problems
2.1 Linear problems

Exercise 12. Prove the following inequality :


Z Z
1 2
2
u(x) dx 6 4 k∇uk2 dx
RN kxk RN

for all N > 3 and for all u ∈ H 1 (RN )

Remark : v
u n 2
uX
N
In that exercise, the norm k · k denotes the euclidean norm on R , i.e. kxk = t xi .
i=1

23
Solution. Step 1 : Let prove the result for u ∈ C0∞ (RN ). Let take u ∈ C0∞ (RN ).
We can write Z +∞
2
u (x) = − 2u(tx)∇u(tx).xdt
1

because an antiderivative of 2u(tx)∇u(tx).x is u2 (tx) and when t → +∞, u(tx) → 0 since u ∈


C0∞ (RN ).
Then, we have
Z
1 2
2
u (x)dx
RN kxk
Z Z +∞
2
= 2
u(tx)∇u(tx).xdt dx
RN kxk 1
Z Z +∞
2
6 2
|u(tx)|k∇u(tx)kkxkdtdx by Cauchy Schwarz
RN kxk 1
Z +∞
|u(tx)|
Z
6 2 k∇u(tx)kdtdx
R N 1 kxk
Z +∞
|u(tx)|
Z
6 2 t k∇u(tx)kdtdx because t > 0
RN 1 ktxk
Z +∞ Z
|u(tx)|
6 2t k∇u(tx)kdxdt by Fubini
1 RN ktxk
Z +∞ Z
|u(y)| 1
6 2t k∇u(y)k N dydt by the change of variable y = tx
1 R N kyk t
Z +∞ 1/2 Z 1/2
|u(y)|2
Z
2 2
6 dt dy k∇u(y)k dy by Cauchy-Schwarz
1 tN −1 RN kyk
2
RN

It follows that
1/2 +∞ 1/2
|u(y)|2
Z Z  Z
2 2
dy 6 dt k∇u(y)k dy
RN kyk2 1 tN −1 RN
Z +∞  +∞
2 2 2
And dt = = 6 2 because n > 3.
1 tN −1 (2 − N )tN −2
1 N −2
Finally, while passing to the square, we find

|u(y)|2
Z Z
2
dy 6 4 k∇u(y)k2 dy
R N kyk RN

Step 2 : The general case.


By density of C0∞ (RN ) in H 1 (RN ) for the H 1 −norm, for all u ∈ H 1 (RN ), there exists a sequence
(un )n ∈ C0∞ (RN )N such that
H 1 (RN )
un −−−−−→ u
n→∞

In other words,
L2 (RN ) L2 (RN )
un −−−−−→ u and ∇un −−−−−→ ∇u
n→∞ n→∞

24
We have, for all n ∈ N,
Z Z
1
un (x)2 dx 6 4 k∇un (x)k2 dx (14)
RN kxk2 RN
| {z }
=k∇un k2 2
L (RN )

We use Fatou’s lemma (because the functions we use are non negative) to have
Z Z
1 2 1
2
u( x) dx = lim inf un (x)2 dx
RN kxk RN kxk2
Z
1
6 lim inf u (x)2 dx
2 n
R N kxk
6 4 lim inf k∇un k2L2 (RN ) by (14)
6 4k∇uk2L2 (RN )
Z
64 k∇u(x)k2 dx
RN

So we have the result for all u ∈ H 1 (RN ).

1
Exercise 13. Let n = 2, 2 < α < 1. Let
n πo
Ωα = (r, θ), 0 < r < 1, 0 < θ <
α
and
u(r, θ) = (r−α − rα ) sin(αθ)

1) Show that ∃q ∗ > 1 such that ∀q ∈ [1, q ∗ [, u ∈ W 1,q (Ωα ).

2) Calculate −∆u

3) What’s up ?

25
y

Ωα

π/α
x
O 1

2
Solution. 1) Step 1 : We want to show that u ∈ Lq (Ωα ) for q < .
α

π q
Z Z Z 1
q
α
q 1 α
|u| = | sin(αθ)| dθ −r r dr
Ωα 0 0 rα
| {z } | {z }
<∞ 1

r∼0 r αq−1

1 2
The function r 7→ is integrable on ]0, 1[ if and only if αq − 1 < 1, i.e. q < .
rαq−1 α
2
Hence, u ∈ Lq (Ωα ) for q < .
α

∂ ∂ 2
Step 2 : We want to show that u, u ∈ Lq (Ωα ) for q < .
∂x ∂y α+1
While deriving u, we have

u = sin(αθ)(−αr−α−1 − αrα−1 ) (15)
∂r

u = α cos(αθ)(r−α − rα ) (16)
∂θ

But
∂ ∂ ∂

 u = cos(θ) u + sin(θ) u

∂r ∂x y
1 ∂ ∂ ∂

 u = − sin(θ) u + cos(θ) u
r ∂θ ∂x ∂y
∂ 1 ∂ ∂ ∂
Hence, if u and u are in Lq , then u, u will be in Lq .
∂r r ∂θ ∂x ∂y

26

q π q
Z Z Z 1
∂ α 1
u = |α sin(αθ)|q dθ + rα−1 r dr
Ωα ∂r rα+1
|0 {z } 0
| {z }
<∞ 1

r∼0 r(α+1)q−1

1
The function r 7→ (α+1)q−1 is integrable on ]0, 1[ if and only if (α + 1)q − 1 < 1, i.e.
r
2
q< .
α+1

q π q
Z Z Z 1
1 ∂ α
q 1 1 α
u = |α cos(αθ)| dθ − r r dr
Ωα r ∂θ rq rα
|0 {z } 0
| {z }
<∞ 1

r∼0 r (α+1)q−1

1
The function r 7→ (α+1)q−1 is integrable on ]0, 1[ if and only if (α + 1)q − 1 < 1, i.e.
r
2
q< .
α+1
∂ ∂ 2
It follows that u, u ∈ Lq (Ωα ) for q < .
∂x ∂y α+1
2
We take q ∗ = to have u ∈ W 1,q (Ωα ) for all q ∈ [1, q ∗ [.
α+1

∂2 1 ∂ 1 ∂2
2) We have ∆u = u + u + u.
∂r2 r ∂r r2 ∂θ2
And we can compute

∂2 −α−2 α−2

u = sin(αθ α(α + 1)r − α(α − 1)r while deriving (15)
∂r2
1 ∂
u = sin(αθ)(−αr−α−2 − αrα−2 ) while using (15)
r ∂r
1 ∂2
u = −α2 sin(αθ)(r−α−2 − rα−2 ) while deriving (16)
r2 ∂θ2

While computing the laplacian, we find that ∆u = 0.

3) It follows that the PDE (


−∆v = 0
v|∂Ωα = 0

has two solutions in W01,q (Ωα ) : u and the zero function.


The problem is that Ωα is not of class C 1 because at the point (0, 0) we have an angle. Then
we lose the uniqueness of the solution.

27

You might also like